Gedankenexperiment mit Quantenverschränkung in Position und ihren Auswirkungen

Stellen Sie sich vor, wir haben zwei Atome A Und B . Sie sind in Position und Impuls miteinander verschränkt, wobei sie durch eine Wellenfunktion im Positionsraum beschrieben werden Ψ ( X A , X B ) . Diese Initialisierung des verschränkten Zustands wird wie in diesem Artikel beschrieben erreicht: http://arxiv.org/abs/quant-ph/9907049

Wir können beide Atome einfangen, indem wir sie mit Lasern in ein harmonisches Oszillatorpotential bringen und sie dazu bringen, beide mit Methoden zu verschränken, die in dem Artikel beschrieben sind, auf den ich oben verwiesen habe, und indem wir einen nicht entarteten optisch-parametrischen Verstärker (NOPA) als Verschränkungsquelle verwenden.

Soweit ich weiß, können wir uns vorstellen, dass jedes Teilchen seine eigene lokale Achse hat und durch einen willkürlichen globalen Abstand getrennt ist. Betrachten wir auch nur eine Dimension, die X -Dimension, um die Dinge einfach zu halten.

Angenommen, wir wenden den Positionsoperator an, X ^ A zu Teilchen A und da es sich in einem harmonischen Oszillatorpotential befand, v , erhalten wir eine einzelne Messung X A . Wenn wir identische oder so nah wie möglich vorbereitete Systeme hätten und denselben Operator oder dieselbe Messung mehrere Male auf ein Ensemble verschränkter Paare anwenden und alle diese Messungen zusammenfügen würden, dann würden wir die Wahrscheinlichkeitsverteilung erhalten oder der "Quantenzustand der Position" des Atoms A . Im Wesentlichen betreiben wir Quantenzustandstomographie. Unter Bezugnahme auf die Abbildung, die ich angehängt habe, habe ich mir ein Szenario ausgedacht, in dem sich das Atom zufällig dazwischen befindet X = -4 und 4 mit willkürlichen Einheiten in Bezug auf ihre lokale Achse.

Geben Sie hier die Bildbeschreibung ein

Atome A Und B sollte ungefähr die gleiche Verteilung haben ... weil jede einzelne Positionsmessung für jedes verschränkte Paar im Ensemble ergeben sollte X A = X B in Bezug auf ihre lokalen Achsen ... denn das bedeutet es, verschränkt zu sein. (Benötigen Sie hier einen Scheck)

Betrachten wir nun zwei Ensembles von Atomen, die wir nennen A Und B . Sie sind in Labor A und Labor B getrennt. Sie sind so verschränkt, dass wir die gleiche Verteilung erhalten, wenn die Position in einem der beiden Labore gemessen wird.

Wir wissen, dass wir bei einem bestimmten Potential eine charakteristische Verteilung erhalten, wenn wir das Ensemble messen. Grundsätzlich gilt: Operator anwenden, X ^ , und Potenzial, v 0 , erzeugt eine andere Verteilung als X ^ , und Potenzial, v 1 , was auch immer diese Potenziale vielleicht sind. (Benötigen Sie hier einen Scheck)

Wir wissen auch, dass wir, wenn wir uns bewerben X ^ , und Potenzial, v 0 + v 1 = v 2 , wird es auch eine andere Verteilung erzeugen, anders als X ^ , und Potenzial, v 0 , oder X ^ , und Potenzial, v 1 , allein. (Benötigen Sie hier einen Scheck)

Wissenschaftler im Labor A hat Ensembles A 1 , A 2 , A 3 , . . . und ein Wissenschaftler im Labor B hat Ensembles B 1 , B 2 , B 3 , . . . . Ensemble A 1 ist verstrickt mit B 1 , A 2 mit B 2 , A 3 mit B 3 , usw.

Wissenschaftler A beschließt, sich zu bewerben v 0 Zu A 1 in ihrem "lokalen" Bereich, Beachten Sie, dass dies tatsächlich gilt Ψ ( X A , X B ) , weil das Ensemble verschränkt ist. Wissenschaftler A überspringt Ensemble A 2 und bewirbt sich v 0 Zu A 3 usw. Wissenschaftler B wendet sich an v 1 blind an alle Ensembles B 1 , B 2 , B 3 , . . . gleichzeitig mit synchronisierten Uhren im Voraus vereinbart

Der Verteilungswissenschaftler B sollte die untere rechte Seite der folgenden Abbildung erhalten: (Beachten Sie, dass ich die Verteilungsformen erfunden habe. Sie werden nur als unterschiedlich gezeigt, um den Fall zu verdeutlichen oder das Gedankenexperiment zu erklären.)

Geben Sie hier die Bildbeschreibung ein

Die logische Schlussfolgerung auf der Grundlage meines vorherigen Verständnisses und Aufbaus ist, wenn alles konsistent ist, dass man eine Nachricht senden könnte, indem man Ensembles verschränkter Atome, synchronisierter Uhren und gleichzeitiger Messungen zu einer bestimmten, im Voraus vereinbarten Zeit verwendet.

Wenn Wissenschaftler A eine Nachricht an Wissenschaftler B senden möchte, würde er oder sie auswählen, welche Ensembles angewendet werden sollen v 0 zu und dann würde sich Wissenschaftler B blind bewerben v 1 unabhängig davon, was Wissenschaftler A getan hat. Wissenschaftler B würde wissen , was Wissenschaftler A gesendet hat, weil beide ihre jeweiligen Potenziale gleichzeitig auf dasselbe gemeinsame oder verflochtene System anwenden, Ψ ( X A , X B ) was zu charakteristischen Verteilungen führt, die "1" "0" "1" zugeordnet werden können.

Hat dieses vorgeschlagene Gedankenexperiment einen großen Fehler? Wenn ja wo? (Der erste Verstoß ist offensichtlich der FTL-Informationsaustausch. Es scheint dagegen zu verstoßen, aber wo?)

Es ist nicht klar, ob Sie das berücksichtigen ( A ich , B ich ) als mehrere unterschiedliche verschränkte Zustände (einer für jeden ich ), wenn Sie an einen global verschränkten Zustand mit totaler Korrelation zwischen den Zuständen denken A ich Und B ich , für jede ich
Sie haben völlig missverstanden, was Verschränkung überhaupt ist und welchen Einfluss ein Potenzial auf einen verschränkten Zustand hat.

Antworten (3)

Sie haben die Auswirkungen des Hinzufügens eines Potentials zu einem verstrickten Zustand völlig missverstanden.

Die Atome a und b sollten ungefähr die gleiche Verteilung haben ... weil jede einzelne Positionsmessung für jedes verschränkte Paar im Ensemble ergeben sollte X A = X B in Bezug auf ihre lokalen Achsen ... denn das bedeutet es, verschränkt zu sein. (Benötigen Sie hier einen Scheck)

Ich möchte hier nur darauf hinweisen, dass die Verstrickung nicht notwendigerweise der Fall istbedeutet, dass die beiden Verteilungen gleich sind. Sie könnten tatsächlich 2 vollkommen identische Zustände haben, die verschränkt sind, genauso wie Sie 2 vollkommen identische Zustände haben könnten, die nicht verschränkt sind. Außerdem könnten Sie auch 2 völlig unterschiedliche Zustände haben und sie verschränken. Was Verschränkung bedeutet, ist, dass Sie feststellen werden, dass es eine Korrelation zwischen den 2 Messungen gibt, nicht dass die 2 eine ähnliche Verteilung haben. Nehmen wir ein einfaches Beispiel, das nicht einmal das geringste bisschen Mathematik erfordert, um es zu verstehen. Stellen Sie sich ein Teilchen mit Spin 0 vor, das in 2 Teilchen mit Spin 1/2 zerfällt. Die Zustände, die diese 2 Teilchen beschreiben, sind jetzt verschränkt, denn was auch immer der Spin ist, der andere muss das Gegenteil sein, damit die Gesamtsumme 0 ist. Beachten Sie, dass beide die gleiche Verteilung haben. Mit anderen Worten, wenn ich bei beiden eine Quantentomografie machen würde, Ich würde feststellen, dass sie beide die Hälfte der Zeit nach oben und die Hälfte der Zeit nach unten drehen. Aber das ist nicht der Grund, warum ich sage, dass sie verstrickt sind. Sie sind verschränkt, weil bei jeder Messung die Zustände beider korreliert sind. Mit anderen Worten, sie sind verschränkt, weil die Messung des Spins eines Teilchens den Spin eines anderen impliziert. Dies ist auf den physikalischen Mechanismus zurückzuführen, der diese 2 Teilchen so erzeugt hat, dass die Gesamtsumme ihrer Spins 0 sein muss. Mit anderen Worten, wenn eines Spin 1/2 ist, ist das andere Spin -1/2 und umgekehrt. Es ist wichtig, diesen Mechanismus im Hinterkopf zu behalten, um den nächsten Fehler zu verstehen, den Sie gemacht haben. wieder verschränkt, weil die Messung des Spins eines Teilchens den Spin eines anderen impliziert. Dies ist auf den physikalischen Mechanismus zurückzuführen, der diese 2 Teilchen so erzeugt hat, dass die Gesamtsumme ihrer Spins 0 sein muss. Mit anderen Worten, wenn eines Spin 1/2 ist, ist das andere Spin -1/2 und umgekehrt. Es ist wichtig, diesen Mechanismus im Hinterkopf zu behalten, um den nächsten Fehler zu verstehen, den Sie gemacht haben. wieder verschränkt, weil die Messung des Spins eines Teilchens den Spin eines anderen impliziert. Dies ist auf den physikalischen Mechanismus zurückzuführen, der diese 2 Teilchen so erzeugt hat, dass die Gesamtsumme ihrer Spins 0 sein muss. Mit anderen Worten, wenn eines Spin 1/2 ist, ist das andere Spin -1/2 und umgekehrt. Es ist wichtig, diesen Mechanismus im Hinterkopf zu behalten, um den nächsten Fehler zu verstehen, den Sie gemacht haben.

Wissenschaftler A beschließt, sich zu bewerben v 0 Zu A 1 in ihrem "lokalen" Bereich, Beachten Sie, dass dies tatsächlich gilt ψ ( X A , X B ) , weil das Ensemble verschränkt ist.

Das ist technisch richtig, aber es bedeutet nicht, was Sie denken, dass es bedeutet. Sicher bewerben v 0 im Labor A beeinflusst ψ ( X A , X B ) , aber nur, weil Teilchen A Teil dieses Systems ist. Tatsächlich ist es auch ein Teil von ψ ( X A , X C ) wobei C ein beliebiges anderes Teilchen im Universum ist, aber wiederum nur, weil Teilchen A Teil dieses Systems ist. Das bedeutet nicht, dass es notwendigerweise Partikel B betrifft, selbst wenn sie verschränkt sind. Mit anderen Worten, wenn ψ ( X A ) Änderungen, dann natürlich ψ ( X A , X B ) verändert sich, weil es ein Teil davon ist. Aber das bedeutet nicht ψ ( X B ) Änderungen. Tatsächlich tut es das nicht. Warum sollte es? Denken Sie daran, dass die beiden Teilchen verschränkt sind, weil ein physikalischer Mechanismus sie verschränkt hat . Um die Bedeutung davon zu verstehen, gehen wir zurück zu unserem Spin-Beispiel.

Erinnern Sie sich zunächst daran, dass, weil die 2 verschränkten Teilchen implizieren, dass, wenn wir A als Spin 1/2 messen, B Spin -1/2 sein muss und umgekehrt, so dass der Gesamtspin 0 ist. Nehmen wir nun an, wir haben a angewendet Potenzial für Teilchen A, das 1 zu seinem Spin hinzugefügt hat (ob dies leicht machbar ist oder nicht, ist hier irrelevant, erwägen Sie einfach die Hypothese). Warum um alles in der Welt sollte das Teilchen B beeinflussen? Du denkst vielleicht "naja, weil es auch den Spin von Teilchen B verschieben würde, da sie verschränkt sind!". Das würde es nicht, und hier ist der Grund. Denken Sie noch einmal daran, dass A und B verschränkt sind, weil es einen physikalischen Grund dafür gibt, dass der Gesamtspin 0 ist. Wenn ein anderer physikalischer Mechanismus den Spin von Teilchen A verschoben hat, v 1 in Ihrem Beispiel) war die Verschiebung der Korrelation. Nun ist es so, dass, wenn ich A als Spin 3/2 gemessen habe, dann B Spin -1/2 sein muss, und wenn A Spin 1/2 ist, dann muss B Spin 1/2 sein. Und so haben wir, wie gesagt, nur die Korrelation verschoben. Denken Sie daran, dass Verschränkung nicht mehr oder weniger als diese Korrelation ist. Und jetzt gilt die übliche Begründung, warum dies keine FTL-Kommunikation zulässt. B weiß nicht, ob A ein Potential angelegt hat oder nicht, bis er eine traditionelle (Lichtgeschwindigkeits-) Kommunikation verwendet, um die Messungen von A zu betrachten. Die Verteilung des Spins für B bleibt in beiden Fällen genau gleich, ebenso für jedes andere Potential wird auf A angewendet. Ein solches Potential betrifft nur den VerwandtenWert der 2 Verteilungen, aber es ändert nicht wirklich Verteilung B. Die gleiche Argumentation würde für ein Potential gelten, das die Position eines räumlich verschränkten Zustands verschieben würde.

Hoffe das hilft, bei Fragen einfach melden.

Ihr Gedankenexperiment hat einen großen Fehler. Gemäß der Quantenmechanik hat bei jeder Messung von zwei räumlich getrennten Atomen a und b, was mit b passiert, absolut keinen Einfluss auf die Wahrscheinlichkeiten der Messergebnisse an b. Ich werde nicht genau herausfinden, was der Fehler in Ihrem vorgeschlagenen Experiment ist, sondern nur angeben, warum die Quantentheorie die FTL-Kommunikation ausschließt.

Bevor ich das tue, werde ich eine genauere Beschreibung der Verschränkung geben, als Sie gegeben haben. Was muss man erklären, um zu verstehen, was in einem Verschränkungsexperiment vor sich geht? Die Probleme sehen oft in etwa so aus.

(1) Es gibt Observables auf A und B, nennen Sie sie Acorr, Bcorr, so dass Sie das finden werden, wenn Sie die Ergebnisse der Messungen vergleichen, nachdem sie abgeschlossen sind und die Informationen über die Messergebnisse an denselben Ort übertragen wurden sie sind korreliert. Wenn Sie also den Elektronenspin messen, kann es sein, dass die Spins mit Wahrscheinlichkeit 1 entgegengesetzt sind oder mit einer gewissen Wahrscheinlichkeit, die sich von 1/2 unterscheidet, wenn sie verglichen werden.

(2) Es gibt Observables auf A und B, nennen Sie sie Anoncorr, Bnoncorr, so dass Sie das finden werden, wenn Sie die Ergebnisse der Messungen vergleichen, nachdem sie abgeschlossen sind und die Informationen über die Messergebnisse an denselben Ort übertragen wurden sie sind nicht korreliert. Wenn Sie also den Elektronenspin messen, kann es sein, dass die Spins mit Wahrscheinlichkeit 1/2 entgegengesetzt und mit Wahrscheinlichkeit 1/2 gleich sind.

(3) Es gibt Zwischenfälle. Und insgesamt sind die Korrelationen so, dass sie nicht mit dem übereinstimmen, was Sie erhalten würden, wenn Sie zwei Systeme hätten, die durch lokale klassische stochastische Variablen repräsentiert würden.

Bei der Bewertung der Ergebnisse eines bestimmten Experiments kann es aufgrund von Messfehlern oder was auch immer zu Komplikationen kommen. Sie können sich auch Formeln ausdenken, wie genau sich die Übereinstimmungswahrscheinlichkeiten unterscheiden, je nachdem, welche Observablen Sie auswählen. Das grundlegende Problem besteht darin, dass die Wahrscheinlichkeit, Korrelationen zu sehen, wenn Sie die Ergebnisse vergleichen, von dem abweicht, was Sie von einer lokalen Theorie erwarten würden, die klassische stochastische Variablen verwendet. Experimente finden solche Korrelationen, aber aus diesen Korrelationen folgt nicht, dass die Quantenmechanik nicht-lokal ist. Egal, welche Messung Sie an System b vornehmen, Sie können nicht sagen, was an System a gemacht wurde.

Was passiert, wenn b gemessen wird? Die Messapparatur unterscheidet mehrere Versionen, die jeweils eines der möglichen Ergebnisse aufgezeichnet haben. Die Messaufzeichnungen auf a werden mit den Messaufzeichnungen auf b korreliert, wenn die Messergebnisse verglichen werden, weil die dekohärenten Systeme, die die Messergebnisse tragen, auch lokal unzugängliche Informationen tragen. Ihre Observablen hängen davon ab, was Bob gemessen hat, aber die Erwartungswerte dieser Observablen hängen nicht von der Messung ab, siehe

http://xxx.lanl.gov/abs/quant-ph/9906007

http://arxiv.org/abs/1109.6223 .

Diese Informationsübertragungen finden vollständig lokal statt und können auf keine andere Weise erfolgen, da die zugrunde liegenden Bewegungsgleichungen lokal sind. Die Quantenmechanik ist nicht nicht-lokal, sie schließt Nicht-Lokalität aus. Die Leute sind manchmal verwirrt darüber, weil sie an etwas glauben, das Zusammenbruch der Wellenfunktion genannt wird. Sie denken, dass bei einer Messung nur ein Ergebnis eintritt, aber in Wirklichkeit treten alle Ergebnisse auf und jede Version des Beobachters sieht nur eines davon als Ergebnis der Dekohärenz. Ihr Experiment wird keine FTL-Kommunikation erzeugen, da Quantensysteme von lokalen Bewegungsgleichungen bestimmt werden.

Sie beschreiben ein Kommunikationsprotokoll, das Setups vom EPR/Bell-Typ nutzt. Wenn ich Ihr Protokoll richtig verstehe, stellen Sie sich einen „Strom“ verschränkter Ensembles vor, und vorab vereinbarte Messungen, die an jedem Block des Stroms durchgeführt werden, um ein „Bit“ zu ergeben. Es gibt verschiedene Möglichkeiten, wie wir dies tun könnten: Sie verwenden das harmonische Oszillatorpotential. Bußgeld. Wir könnten den Klassiker machen X vs z Spin oder verschiedene andere Messungen.

Sie fragen, wo der FTL-Informationsaustausch stattfindet. Ich gehe davon aus, obwohl Sie dies nicht zu spezifizieren scheinen, dass Ihre beiden Parteien räumlich getrennt sind (eine stärkere und spezifischere Einschränkung als nur „getrennt“ zu sein) und dass die Geschwindigkeit der „Übertragung“ so ist, dass dies der Fall ist impliziert Nichtlokalität. Letztendlich reduziert sich die Analyse Ihres Protokolls auf eine Standardanalyse von Bell-Experimenten.

Dies wird auf Ihre gewählte Interpretation der Quantenmechanik hinauslaufen. Dies ist keine endgültige Debatte, aber einige Positionen sind eindeutig haltbarer als andere. In einer „naiven“ Kopenhagener Interpretation gibt es einen FTL-Informationsaustausch. Dies ist der springende Punkt des Einstein-Podolsky-Rosen-Paradoxons (ihr Argument war, dass QM nicht vollständig sein könne, weil die Verletzungen des lokalen Realismus physikalisch inakzeptabel seien). Bell baute darauf auf, indem er zeigte, dass es nicht durch lokale verborgene Variablen erklärt werden konnte; Wir können Bohm an dieser Stelle wahrscheinlich außer Acht lassen. Alanf gab eine gute – und schrille – Analyse von einer Position ab, die er von Everett (+1) geerbt hatte. Ich werde mir die relationale Interpretation ansehen .

Dekohärenz wirft Probleme für Ihr Protokoll auf, ähnlich wie für viele Ideen vom Typ Quanteninformation. Der Kampf besteht darin, einen Weg zu finden, Ihre Systeme zu isolieren, damit sie nicht auseinanderfallen. Nehmen wir an, Sie haben dies getan (eine nicht triviale Aufgabe).

Wir haben dann das Problem zu verifizieren, dass der Informationsaustausch die Integrität bewahrt hat, dh dass das, was B erhält, tatsächlich dem entspricht, was A gesendet hat. Wie machen wir das? Nun, sie müssten sich „treffen“: nicht unbedingt persönlich, aber ihre Lichtkegel müssten sich in irgendeiner Form schneiden; E-Mail genügt. Daher ist es unmöglich, mit Sicherheit zu sagen, dass Ihr Informationsaustausch mit Integrität durchgeführt wurde, bis wieder lokale Beschränkungen gelten. Und wie von Zauberhand stellen wir fest, dass dies der Fall ist; A und B werden zustimmen. Wir könnten argumentieren, dass wir könntenGewissheit haben, wenn wir akzeptieren, dass unsere Theorie die Welt angemessen wiedergibt; Aber der Punkt ist, dass wir die Theorie nicht wirklich verifizieren können, bis wir über einen alternativen Kanal bestätigt haben, dass unsere Ergebnisse das sind, was die Theorie vorhersagt (oder sie falsifizieren die Theorie oder bestätigen oder was auch immer Ihre Wissenschaftsphilosophie sagt). der beste Weg, es anzugehen).

Das soll nicht heißen, dass wir die Gültigkeit der zugrunde liegenden Theorie anzweifeln sollten oder dass wir Grund zu der Annahme haben, dass die Messungen aus irgendeinem Grund nicht übereinstimmen würden (im Gegenteil, mehrere Bell-Experimente deuten darauf hin, dass dies der Fall ist, und Zeilingers Arbeit erweiterte dies auf Drei-Wege-GHZ-Verschränkungen). Vielmehr impliziert es stark, dass die zugrunde liegende Theorie uns etwas über lokalen Realismus sagt ... dh dass es unphysikalisch ist, Quantensystemen bei raumartiger Trennung objektiv bestimmte (nicht relationale) Zustände zuzuschreiben; sie müssen „in Bezug auf“ ein anderes System angegeben werden. Der Kürze halber dieses Papiergeht viel detaillierter auf die relationale Analyse von EPR ein. Es gibt eine sehr hypothetische Erklärung im Stil eines Gedankenexperiments, aber eine, die (meiner Meinung nach) analytisch gültig ist, ohne „spukhafte Fernwirkung“. Dies schließt die Idee der Dekohärenz nicht aus, sondern eher eine andere Interpretation davon an der Grenze; Die meiste Dekohärenz geht nicht einmal auf diese Art von Szenarien ein, sondern ist eher der Grund, warum der Mond da ist, wenn ich nicht hinschaue; Quantensysteme geben Informationen in die Welt ab. Der Unterschied zwischen dieser und Everett-basierten Interpretationenist das, wo sie sagen „alle Zustände, eine einzige Beschreibung“, die relationale Interpretation sagt „ein einziger Zustand relativ zu einem einzelnen Beobachter, und letztendlich werden alle kohärent sein“. Wählen Sie Ihr Gift - das Problem ist nicht geklärt, und dies ist wahrscheinlich kein Forum, um darüber zu streiten, was besser ist (Philosophie, wirklich, nicht Physik, da sie empirisch nicht unterschieden werden konnten).

Daher ist das FTL-Problem nicht eindeutig. Es kann sein, dass es passiert, aber die meisten modernen Interpretationen würden argumentieren, dass es nicht so ist. Aber es wurde sicherlich experimentell gezeigt, dass EPR/Bell-Phänomene auftreten.

Zusammenfassend: Stimmt Ihr vorgeschlagenes Experiment mit EPR/Bell-Szenarien zur Informationsübertragung überein? Ja, im Prinzip, wenn Sie Messungen konstruieren könnten, die Ihre Anforderungen erfüllen v 0 + v 1 = v 2 Kriterien. Es gibt ohnehin andere Messungen, die im Wesentlichen dasselbe Protokoll ermöglichen würden, dh B führt eine konstante Messung durch, deren Ergebnisse durch eine von A gewählte Messung beeinflusst zu sein scheinen (z. B. misst B immer Spin Along z , und A misst den Spin in einem anderen Winkel nur für diese Ensembles A 1 , A 2 , A 3 er möchte '1'-Bits sein, also unterscheidet sich die Verteilung von B für jedes Ensemble in Abhängigkeit davon, ob A gemessen wurde oder nicht, da der Gesamtspin jedes Singulett-Paares immer 0 ist). Bedeutet es FTL-Kommunikation? Hängt von Ihrer Interpretation der Quantenmechanik ab, aber es gibt keinen Grund, warum es so sein sollte.

Sein Fehler ist ein Fehler, unabhängig von der von Ihnen gewählten Interpretation von QM.